The gradient is m = &#x2212;<!-- − --> 4 / 3 , and the point given is ( 3

Marissa Singh

Marissa Singh

Answered question

2022-05-09

The gradient is m = 4 / 3, and the point given is ( 3 / 2 , 3 )
Let c be the y-intercept
y = m x + c
y = ( 4 / 3 ) x + c
If I substitute x and y from the point given, 3 = ( 4 / 3 ) ( 3 / 2 ) + c and look for c, I get c = 5
But according to my answer sheet, c = 33 / 8 by using y y 1 = m ( x x 1 )
Why are the two methods giving different answers?

Answer & Explanation

Marquis Matthews

Marquis Matthews

Beginner2022-05-10Added 9 answers

The answer sheet is wrong. Using y y 1 = m ( x x 1 ), we get y 3 = ( 4 / 3 ) ( x 3 / 2 ). Distributing the 4 / 3, we get y 3 = ( 4 / 3 ) x + 4 / 2 = ( 4 / 3 ) x + 2. Now add three to both sides, and we get y = ( 4 / 3 ) x + 5, so c = 5 again.
Alaina Holt

Alaina Holt

Beginner2022-05-11Added 3 answers

It looks like someone flipped the gradient at some point. If you use 3 4 as the gradient, you end up with c = 33 8 .

Do you have a similar question?

Recalculate according to your conditions!

New Questions in High school statistics

Ask your question.
Get an expert answer.

Let our experts help you. Answer in as fast as 15 minutes.

Didn't find what you were looking for?